If $Ntrianglelefteq G$, then $phi(N) leq phi(G)$, where $phi(N)$ is the Frattini subgroup of $N$.












3












$begingroup$


I was thinking somehow to use normality of N as follows



Since N is normal, then $G/N$ will be a group, so we can consider the
natural map



$pi : G rightarrow G/N$, where $g mapsto gN$, $ker(pi) = N$.



So It is here enough to prove that a maximal subgroup M of G will contain
$ker(pi)$, then since $phi(G)$ is intersection all such maximal subgroup M, so we will get that $phi(N) leq phi(G)$, but I got stuck in this specific step.










share|cite|improve this question











$endgroup$

















    3












    $begingroup$


    I was thinking somehow to use normality of N as follows



    Since N is normal, then $G/N$ will be a group, so we can consider the
    natural map



    $pi : G rightarrow G/N$, where $g mapsto gN$, $ker(pi) = N$.



    So It is here enough to prove that a maximal subgroup M of G will contain
    $ker(pi)$, then since $phi(G)$ is intersection all such maximal subgroup M, so we will get that $phi(N) leq phi(G)$, but I got stuck in this specific step.










    share|cite|improve this question











    $endgroup$















      3












      3








      3


      1



      $begingroup$


      I was thinking somehow to use normality of N as follows



      Since N is normal, then $G/N$ will be a group, so we can consider the
      natural map



      $pi : G rightarrow G/N$, where $g mapsto gN$, $ker(pi) = N$.



      So It is here enough to prove that a maximal subgroup M of G will contain
      $ker(pi)$, then since $phi(G)$ is intersection all such maximal subgroup M, so we will get that $phi(N) leq phi(G)$, but I got stuck in this specific step.










      share|cite|improve this question











      $endgroup$




      I was thinking somehow to use normality of N as follows



      Since N is normal, then $G/N$ will be a group, so we can consider the
      natural map



      $pi : G rightarrow G/N$, where $g mapsto gN$, $ker(pi) = N$.



      So It is here enough to prove that a maximal subgroup M of G will contain
      $ker(pi)$, then since $phi(G)$ is intersection all such maximal subgroup M, so we will get that $phi(N) leq phi(G)$, but I got stuck in this specific step.







      abstract-algebra






      share|cite|improve this question















      share|cite|improve this question













      share|cite|improve this question




      share|cite|improve this question








      edited Jan 3 at 20:09









      Dietrich Burde

      81.5k648106




      81.5k648106










      asked Oct 12 '15 at 6:46







      user111750





























          1 Answer
          1






          active

          oldest

          votes


















          0












          $begingroup$

          The statement has been discussed here:



          For Frattini subgroup $Phi(G)$, if $N vartriangleleft G$, then $Phi(N) leq Phi(G)$. : Counter-example



          We have the following result (for finite groups it coincides with the above statement):



          Theorem: The Frattini subgroup of any normal subgroup is contained in the Frattini subgroup of the whole group, provided the normal subgroup is a group in which every proper subgroup is contained in a maximal subgroup.



          Proof: The Frattini subgroup $Phi(N)$ is a characteristic subgroup of $N$. Since every characteristic subgroup of a normal subgroup is normal, $Phi(N)$ is a normal subgroup of G.
          We need to show that $Phi(N)$ is contained in $Phi(G)$. For this, it suffices to show that $Phi(N)$ is contained in every maximal subgroup of $G$. We prove this by contradiction.
          Suppose $M$ is a maximal subgroup of $G$ not containing $Phi(N)$. Then, the subgroup generated by $M$ and $Phi(N)$ is equal to $G$. Since $Phi(N)$ is normal in $G$, we have $MPhi(N) = G$. From that, it follows that
          $Phi(N)(M cap N) = N$.
          Since $M$ does not contain $Phi(N)$, $M$ does not contain $N$ either and hence
          $M cap N$ is a proper subgroup of $N$. Since every proper subgroup is contained in a maximal subgroup of $N$, there is a maximal subgroup of $N$ containing both $M cap N$ and $Phi(N)$. This contradicts the fact that their product is $N$.






          share|cite|improve this answer









          $endgroup$














            Your Answer





            StackExchange.ifUsing("editor", function () {
            return StackExchange.using("mathjaxEditing", function () {
            StackExchange.MarkdownEditor.creationCallbacks.add(function (editor, postfix) {
            StackExchange.mathjaxEditing.prepareWmdForMathJax(editor, postfix, [["$", "$"], ["\\(","\\)"]]);
            });
            });
            }, "mathjax-editing");

            StackExchange.ready(function() {
            var channelOptions = {
            tags: "".split(" "),
            id: "69"
            };
            initTagRenderer("".split(" "), "".split(" "), channelOptions);

            StackExchange.using("externalEditor", function() {
            // Have to fire editor after snippets, if snippets enabled
            if (StackExchange.settings.snippets.snippetsEnabled) {
            StackExchange.using("snippets", function() {
            createEditor();
            });
            }
            else {
            createEditor();
            }
            });

            function createEditor() {
            StackExchange.prepareEditor({
            heartbeatType: 'answer',
            autoActivateHeartbeat: false,
            convertImagesToLinks: true,
            noModals: true,
            showLowRepImageUploadWarning: true,
            reputationToPostImages: 10,
            bindNavPrevention: true,
            postfix: "",
            imageUploader: {
            brandingHtml: "Powered by u003ca class="icon-imgur-white" href="https://imgur.com/"u003eu003c/au003e",
            contentPolicyHtml: "User contributions licensed under u003ca href="https://creativecommons.org/licenses/by-sa/3.0/"u003ecc by-sa 3.0 with attribution requiredu003c/au003e u003ca href="https://stackoverflow.com/legal/content-policy"u003e(content policy)u003c/au003e",
            allowUrls: true
            },
            noCode: true, onDemand: true,
            discardSelector: ".discard-answer"
            ,immediatelyShowMarkdownHelp:true
            });


            }
            });














            draft saved

            draft discarded


















            StackExchange.ready(
            function () {
            StackExchange.openid.initPostLogin('.new-post-login', 'https%3a%2f%2fmath.stackexchange.com%2fquestions%2f1475987%2fif-n-trianglelefteq-g-then-phin-leq-phig-where-phin-is-the-fra%23new-answer', 'question_page');
            }
            );

            Post as a guest















            Required, but never shown
























            1 Answer
            1






            active

            oldest

            votes








            1 Answer
            1






            active

            oldest

            votes









            active

            oldest

            votes






            active

            oldest

            votes









            0












            $begingroup$

            The statement has been discussed here:



            For Frattini subgroup $Phi(G)$, if $N vartriangleleft G$, then $Phi(N) leq Phi(G)$. : Counter-example



            We have the following result (for finite groups it coincides with the above statement):



            Theorem: The Frattini subgroup of any normal subgroup is contained in the Frattini subgroup of the whole group, provided the normal subgroup is a group in which every proper subgroup is contained in a maximal subgroup.



            Proof: The Frattini subgroup $Phi(N)$ is a characteristic subgroup of $N$. Since every characteristic subgroup of a normal subgroup is normal, $Phi(N)$ is a normal subgroup of G.
            We need to show that $Phi(N)$ is contained in $Phi(G)$. For this, it suffices to show that $Phi(N)$ is contained in every maximal subgroup of $G$. We prove this by contradiction.
            Suppose $M$ is a maximal subgroup of $G$ not containing $Phi(N)$. Then, the subgroup generated by $M$ and $Phi(N)$ is equal to $G$. Since $Phi(N)$ is normal in $G$, we have $MPhi(N) = G$. From that, it follows that
            $Phi(N)(M cap N) = N$.
            Since $M$ does not contain $Phi(N)$, $M$ does not contain $N$ either and hence
            $M cap N$ is a proper subgroup of $N$. Since every proper subgroup is contained in a maximal subgroup of $N$, there is a maximal subgroup of $N$ containing both $M cap N$ and $Phi(N)$. This contradicts the fact that their product is $N$.






            share|cite|improve this answer









            $endgroup$


















              0












              $begingroup$

              The statement has been discussed here:



              For Frattini subgroup $Phi(G)$, if $N vartriangleleft G$, then $Phi(N) leq Phi(G)$. : Counter-example



              We have the following result (for finite groups it coincides with the above statement):



              Theorem: The Frattini subgroup of any normal subgroup is contained in the Frattini subgroup of the whole group, provided the normal subgroup is a group in which every proper subgroup is contained in a maximal subgroup.



              Proof: The Frattini subgroup $Phi(N)$ is a characteristic subgroup of $N$. Since every characteristic subgroup of a normal subgroup is normal, $Phi(N)$ is a normal subgroup of G.
              We need to show that $Phi(N)$ is contained in $Phi(G)$. For this, it suffices to show that $Phi(N)$ is contained in every maximal subgroup of $G$. We prove this by contradiction.
              Suppose $M$ is a maximal subgroup of $G$ not containing $Phi(N)$. Then, the subgroup generated by $M$ and $Phi(N)$ is equal to $G$. Since $Phi(N)$ is normal in $G$, we have $MPhi(N) = G$. From that, it follows that
              $Phi(N)(M cap N) = N$.
              Since $M$ does not contain $Phi(N)$, $M$ does not contain $N$ either and hence
              $M cap N$ is a proper subgroup of $N$. Since every proper subgroup is contained in a maximal subgroup of $N$, there is a maximal subgroup of $N$ containing both $M cap N$ and $Phi(N)$. This contradicts the fact that their product is $N$.






              share|cite|improve this answer









              $endgroup$
















                0












                0








                0





                $begingroup$

                The statement has been discussed here:



                For Frattini subgroup $Phi(G)$, if $N vartriangleleft G$, then $Phi(N) leq Phi(G)$. : Counter-example



                We have the following result (for finite groups it coincides with the above statement):



                Theorem: The Frattini subgroup of any normal subgroup is contained in the Frattini subgroup of the whole group, provided the normal subgroup is a group in which every proper subgroup is contained in a maximal subgroup.



                Proof: The Frattini subgroup $Phi(N)$ is a characteristic subgroup of $N$. Since every characteristic subgroup of a normal subgroup is normal, $Phi(N)$ is a normal subgroup of G.
                We need to show that $Phi(N)$ is contained in $Phi(G)$. For this, it suffices to show that $Phi(N)$ is contained in every maximal subgroup of $G$. We prove this by contradiction.
                Suppose $M$ is a maximal subgroup of $G$ not containing $Phi(N)$. Then, the subgroup generated by $M$ and $Phi(N)$ is equal to $G$. Since $Phi(N)$ is normal in $G$, we have $MPhi(N) = G$. From that, it follows that
                $Phi(N)(M cap N) = N$.
                Since $M$ does not contain $Phi(N)$, $M$ does not contain $N$ either and hence
                $M cap N$ is a proper subgroup of $N$. Since every proper subgroup is contained in a maximal subgroup of $N$, there is a maximal subgroup of $N$ containing both $M cap N$ and $Phi(N)$. This contradicts the fact that their product is $N$.






                share|cite|improve this answer









                $endgroup$



                The statement has been discussed here:



                For Frattini subgroup $Phi(G)$, if $N vartriangleleft G$, then $Phi(N) leq Phi(G)$. : Counter-example



                We have the following result (for finite groups it coincides with the above statement):



                Theorem: The Frattini subgroup of any normal subgroup is contained in the Frattini subgroup of the whole group, provided the normal subgroup is a group in which every proper subgroup is contained in a maximal subgroup.



                Proof: The Frattini subgroup $Phi(N)$ is a characteristic subgroup of $N$. Since every characteristic subgroup of a normal subgroup is normal, $Phi(N)$ is a normal subgroup of G.
                We need to show that $Phi(N)$ is contained in $Phi(G)$. For this, it suffices to show that $Phi(N)$ is contained in every maximal subgroup of $G$. We prove this by contradiction.
                Suppose $M$ is a maximal subgroup of $G$ not containing $Phi(N)$. Then, the subgroup generated by $M$ and $Phi(N)$ is equal to $G$. Since $Phi(N)$ is normal in $G$, we have $MPhi(N) = G$. From that, it follows that
                $Phi(N)(M cap N) = N$.
                Since $M$ does not contain $Phi(N)$, $M$ does not contain $N$ either and hence
                $M cap N$ is a proper subgroup of $N$. Since every proper subgroup is contained in a maximal subgroup of $N$, there is a maximal subgroup of $N$ containing both $M cap N$ and $Phi(N)$. This contradicts the fact that their product is $N$.







                share|cite|improve this answer












                share|cite|improve this answer



                share|cite|improve this answer










                answered Jan 3 at 20:15









                Dietrich BurdeDietrich Burde

                81.5k648106




                81.5k648106






























                    draft saved

                    draft discarded




















































                    Thanks for contributing an answer to Mathematics Stack Exchange!


                    • Please be sure to answer the question. Provide details and share your research!

                    But avoid



                    • Asking for help, clarification, or responding to other answers.

                    • Making statements based on opinion; back them up with references or personal experience.


                    Use MathJax to format equations. MathJax reference.


                    To learn more, see our tips on writing great answers.




                    draft saved


                    draft discarded














                    StackExchange.ready(
                    function () {
                    StackExchange.openid.initPostLogin('.new-post-login', 'https%3a%2f%2fmath.stackexchange.com%2fquestions%2f1475987%2fif-n-trianglelefteq-g-then-phin-leq-phig-where-phin-is-the-fra%23new-answer', 'question_page');
                    }
                    );

                    Post as a guest















                    Required, but never shown





















































                    Required, but never shown














                    Required, but never shown












                    Required, but never shown







                    Required, but never shown

































                    Required, but never shown














                    Required, but never shown












                    Required, but never shown







                    Required, but never shown







                    Popular posts from this blog

                    To store a contact into the json file from server.js file using a class in NodeJS

                    Redirect URL with Chrome Remote Debugging Android Devices

                    Dieringhausen